edited by
66 views
0 votes
0 votes
Let $H$ and $K$ be subgroups of a group $G$ of finite indices $(\text{i.e.}, [G: H]<$ $\infty$ and $[G: K]<\infty).$ Prove that $H \cap K$ is also of finite index $(\text{i.e.}, [G: H \cap K]<\infty).$
edited by

Please log in or register to answer this question.

Related questions

0 votes
0 votes
0 answers
1
admin asked Aug 8, 2022
295 views
Let $\left\{a_{n}\right\}$ be a decreasing sequence such that $\displaystyle{}\sum_{n=1}^{\infty} a_{n}$ is convergent. Prove that the sequence $\left\{n a_{n}\right\}$ g...
0 votes
0 votes
1 answer
2
admin asked Aug 8, 2022
102 views
Consider an $n \times n$ matrix $A=I_{n}-\alpha \alpha^{T}$, where $I_{n}$ is the identity matrix of order $n$ and $\alpha$ is an $n \times 1$ column vector such that $\a...
1 votes
1 votes
0 answers
3
admin asked Aug 8, 2022
355 views
Let $A$ and $B$ be two invertible real matrices of order $n$. Show that $\det(x A+(1-x) B)=0$ has finitely many solutions for $x.$
0 votes
0 votes
0 answers
4
admin asked Aug 8, 2022
81 views
Show that for every $\theta \in\left(0, \frac{\pi}{2}\right),$ there exists a unique real number $x_{\theta}$ such that $$ (\sin \theta)^{x_{\theta}}+(\cos \theta)^{x_{\t...